Difference between revisions of "1987 AJHSME Problems"

(Problem 2)
(Problem 3)
Line 16: Line 16:
  
 
== Problem 3 ==
 
== Problem 3 ==
 +
 +
<math>2(81+83+85+87+89+91+93+95+97+99)=</math>
 +
 +
<math>\text{(A)}\ 1600 \qquad \text{(B)}\ 1650 \qquad \text{(C)}\ 1700 \qquad \text{(D)}\ 1750 \qquad \text{(E)}\ 1800</math>
  
 
[[1987 AJHSME Problems/Problem 3|Solution]]
 
[[1987 AJHSME Problems/Problem 3|Solution]]

Revision as of 15:03, 28 January 2009